Answer:
do practice review packets and grade yourself
Step-by-step explanation:
there is a website called escambia and google escambia algebra 1 eoc review and there is every thing you need to pass.
please mark me brainliest
Final answer:
To perform well in the Algebra 1 EOC exam, students should study the topics specified by the instructor using provided tools like study guides and flashcards, engage in self-evaluation and active learning, and maintain a good study routine.
Explanation:
To excel in your Algebra 1 End-of-Course (EOC) exam, it is crucial to have a structured and effective study plan. Begin with a clear understanding of the topics you need to cover, such as algebraic expressions, equations, exponents, and functions.
Utilize tools provided by your instructor, like reviews, study guides, flashcards, and PowerPoints. These are specifically tailored to what your instructor deems important and may include topics like number patterns, finance, growth, trigonometry, analytical geometry, and statistics.
Regular self-evaluation helps identify areas that need more attention. Review quizzes and textbook chapter questions, as they are likely indicative of the questions you'll face during the exam. Additionally, engage in active learning by organizing information into categories and patterns, which helps in understanding and retaining complex concepts.
Suppose Y varies directlly With x. If y= 6 when x=2 find x when y= 12
Answer:
The value of x = 4 when the value of y = 12
Step-by-step explanation:
∵ y α x ⇒ it is direct variation
- It means y increase when x increase
∴ y = kx ⇒ where k is a constant
- To find the value of k we will substitute the values of x
and y in the equation above
- If y = 6 when x = 2
∴ 6 = k(2) ⇒ divide two sides by 2
∴ k = 3
∴ y = 3x ⇒ equation of variation
- To find the value of x when y = 12
- Substitute the value of y in the equation of variation
∴ 12 = 3x ⇒ divide both sides by 3
∴ x = 4
* The value of x = 4 when the value of y = 12
Answer: x=4
Step-by-step explanation:
By definition, the direct variation eqution has the following form:
[tex]y=kx[/tex]
Where k is the constant.
Therefore, if If [tex]y=6[/tex] when [tex]x=2[/tex], you can calculate k as following:
-Substitute values.
- Solve for k.
Then:
[tex]6=k(2)\\k=3[/tex]
Then, when [tex]y=12[/tex], you can calculate the value of x as it is shown below:
-Substitute values.
- Solve for x.
Then:
[tex]12=3x\\x=4[/tex]
52.08 ÷ 3 =
A) 16.18
B) 16.36
C) 17.18
D) 17.36
Hey there!
52.08 ÷ 3 = 17.36
The correct answer is D
Hope this helps you!
God bless ❤️
xXxGolferGirlxXx
I need help with this
Answer:
V = 235.5 in³Step-by-step explanation:
The formula of a volume of a cylinder:
[tex]V=\pi r^2H[/tex]
r - radius
H - height
From the pictue we have r = 5in and H = 3in. Substitute:
[tex]x=\pi(5^2)(3)=\pi(25)(3)=75\pi\ in^3[/tex]
[tex]\pi\approx3.14[/tex]
Therefore
[tex]x=(75)(3.14)=235.5\ in^3[/tex]
Answer:
75π is the answer
Step-by-step explanation:
In term of pie is solving without multiplying with pie which is 3.14
A=πr2 x H
1) leave π alone solve for r2
The radius is 5 so 5 squared is 25 or 5x5=25
2) Just multiply 25 times the height which is 3 with pie
25x3=75
75π is the answer
Hopes this helps!
write the equation 4x+y=7 in slope intercept form
The equation 4x + y = 7 can be rewritten in slope-intercept form as y = -4x + 7, where -4 is the slope and 7 is the y-intercept. This format makes it easier to graph the line as it directly gives us the slope and y-intercept.
Explanation:To rewrite the equation 4x + y = 7 in slope-intercept form (y=mx+b), we need to solve for y. This equation can be transformed by subtracting 4x from both sides, resulting in y = -4x + 7. Here, -4 is the slope (m) and 7 is the y-intercept (b).
In comparison, if we look at the given graphic (Figure A1), we see another equation where the slope or 'm' is 3 and the y-intercept 'b' is 9. This slope and y-intercept gives us the equation y = 3x + 9. Thus, it can be seen that y = mx + b format helps to better understand the geometry of a linear equation on a graph, with 'm' indicating the slope and 'b' being the y-intercept.
Learn more about Slope-Intercept Form here:https://brainly.com/question/29146348
#SPJ3
What is the volume of this cube?
Enter your answer as a decimal in this box.
it would be 25.2 hope this helps
Look at the following math problems. 10 + 2 = 12 12 − 2 = 10 What are these operations called?
Answer:
i think it might be fact families
Step-by-step explanation:
this is because they use the 3 same #rrs
Answer: Inverse Operations
Step-by-step explanation:
Two positive numbers have a sum of 8 and their product is equal to the larger number plus 10
Answer:
There are 2 sets of numbers that work...
x = 3, y = 5
and
x = 6, y = 2
Step-by-step explanation:
Let x be one number, and let y be the other number. We have 2 equations...
x + y = 8 (two positive numbers have a sum of 8)
xy = y + 10 (their product is equal to the larger number plus 10)
solve the first equation for y, and substitute that into the second equation...
y = 8 - x
x(8 - x) = 8 - x + 10
now solve for x...
8x - x² = 18 - x
This is a quadratic, so get everything to one side so it's equal to zero...
-x² + 9x - 18 = 0 (add x and subtract 18 from both sides)
Now solve for x...
x² - 9x + 18 = 0 (divide both sides by -1)
(x - 6)(x - 3) = 0 (factor)
so
x - 6 = 0 becomes x = 6 (add 6 to both sides)
and
x - 3 = 0 becomes x = 3 (add 3 to both sides
If x is 3, then y = 5 (3 + 5 is 8, and 3(5) = 5 + 10, both equations hold up)
If x is 6, then y = 2 (6 + 2 is 8, and 6(2) = 2 + 10, both equation hold up)
Find the measure of a central angle that intercepts an arc of 3 inches in a circle whose radius is 8 inches.
0.375 rad
1.05 rad
2.3 rad
Answer:
option (A)
central angle = 0.375 rad
Step-by-step explanation:
Given in the question,
radius of the circle = 8 inches
arc of the circle = 3 inches
To find,
measure of a central angle
Central angles are subtended by an arc between those two points.
Formula to use:
s = (r) (θ)where r = radius
s = arc length
θ = angle in radians
Plug in the values in the equation
3 = (8) (θ)θ = 3/8 0.375 rad
Answer:
.375 rad
Step-by-step explanation:
Using the equation w=v/r
w=3/8
w=.375
HELP ME PLZ
*****************
What is question 3? Please help me
It would be answer choice (B) :D
Answer:
See below.
Step-by-step explanation:
Problem A
I have no tool that will draw this easily but I can direct you to do it.
Draw 4 bricks separated by 3 spaces on the top row of bricks. They should be going horizontally. Draw 9 bricks going vertically. They should not touch the bricks in the first row.Draw 4 bricks going horizontally separated by 3 spaces just like the ones drawn in step one.Problem B
The general formula for the number of bricks in any pattern (n) is
4n + 1 where n is the pattern number. Try some values.
n =3
4n + 1
4*3 + 2
13
n = 4 which is the one you just drew
4n + 1
4*4 + 1
16 + 1
17
n = 8
4n + 1
4*8 + 1
32 + 1
33
Problem C
4n + 1 = 49 Formula
4n -1 = 49 - 1 Subtract 1 from both sides
4n = 48 Divide by 4
4n/4 = 48/4
n = 12 Answer
Pattern 12 will have 49 bricks
Problem D
The general formula we have been using all along is the answer.
4n + 1
Point O is the center of the circle. What is the value of X?
Answer options: 64, 54, 34, 44.
Answer:
34°
Step-by-step explanation:
The angle made when a tangent and a radius intersect=90°
Therefore angle OQP =90°
The triangle made by the two radii is isosceles ( base angles are equal) hence angle POQ=180-2(62)
=56°
The right triangle OPQ can hence be solved as follows
angle POQ=56°
angle OQP=90°
angle x=180-(56+90)=34° as all the interior angles of any triangle add up to 180°
16 =2/3 f equals what
Answer:
The answer is f = 24
Step-by-step explanation:
In order to find this, multiply both sides by 3/2
16 = 2/3f
24 = f
solve by factoring square roots x^2 -49=0
Answer:
X=7
Step-by-step explanation:
X = SQRT(49)
X = 7
Best regards
what is the shortest time it takes to collect a container of water
The shortest time it takes to collect a container of water is 45 minutes.
What is Box plot?
A box and whisker plot, often known as a box plot, shows a data set's five-number summary. The minimum, first quartile, median, third quartile, and maximum make up the five-number summary. A box is drawn from the first quartile to the third quartile in a box plot. At the median, a vertical line passes through the box.
We have given the box plot which shows the amount water collect in how many minutes.
The box plot gives the following information
Minimum value = 45
Median = 48
Lower Quartile = 53
Upper Quartile = 56
Maximum = 64
So, the shortest time is given by minimum value which is 45 minutes.
Learn more about Box plot here:
https://brainly.com/question/1523909
#SPJ7
Use formula I=prt to solve:
Steve borrowed 450$ for 3 years the yearly interest rate was 13%. How much interest did he owe at the end of 3 years?
Answer: $175.50
Step-by-step explanation: Since you are given the formula, your next step is to put your information into it.
I = PRT
I = 450 x .13 x 3
Now to solve for it:
I = $175.50
What is the length of the side labeled x cm?
Answer:
The length of side [tex]x[/tex] is 4.9 cm
Step-by-step explanation:
The first thing we need to do is find the third angle using the angle sum theorem.
The angle sum theorem states that the sum of the interior angles of a triangle is always 180°. A + B + C = 180°
Let's call our missing angle A, so
A + 41° + 62° = 180°
A + 103° = 180°
A = 180° - 103°
A = 77°
Now we can use the rule of sines to find side [tex]x[/tex]:
[tex]\frac{x}{sin(41)} =\frac{14}{sin(77)}[/tex]
[tex]x=\frac{14sin(41)}{sin(77)}[/tex]
[tex]x=9.4[/tex]
We can conclude that the correct answer is 9.4 cm
A park in the shape of a right triangle is surrounded by three streets. The park is 120 feet along Park Street and 95 feet along Water Street. What is the area of the park?
Area of a triangle
1/2×b×h
=1/2×120×95
=60×95
=5700
Answer
5700 ft.
Answer:
5700 ft
Step-by-step explanation:
Which quantity is proportional to 65⁄5? Check all that are true. 13⁄1 260⁄10 195⁄15 130⁄15 130⁄10
Answer:
[tex]\frac{13}{1}[/tex]
[tex]\frac{195}{15}[/tex]
[tex]\frac{130}{10}[/tex]
Step-by-step explanation:
we have the ratio
[tex]\frac{65}{5}[/tex]
step 1
Divide both sides (numerator and denominator) by 5
[tex]\frac{65/5}{5/5}=\frac{13}{1}[/tex]
therefore
[tex]\frac{13}{1}[/tex] is proportional to [tex]\frac{65}{5}[/tex]
step 2
Multiply both sides (numerator and denominator) by 3
[tex]\frac{65*3}{5*3}=\frac{195}{15}[/tex]
therefore
[tex]\frac{195}{15}[/tex] is proportional to [tex]\frac{65}{5}[/tex]
step 3
Multiply both sides (numerator and denominator) by 2
[tex]\frac{65*2}{5*2}=\frac{130}{10}[/tex]
therefore
[tex]\frac{130}{10}[/tex] is proportional to [tex]\frac{65}{5}[/tex]
What is the following sum? 4(5 sqrt x^2y)+3(5 sqrt x^2y) PLEASE HELP IM TIMED
Answer: [tex]35\sqrt{x^2y}[/tex]
Step-by-step explanation:
To solve this exercise you must apply the proccedure shown below:
- Apply multiplication, as following:
[tex]4(5\sqrt{x^2y})+3(5\sqrt{x^2y})=20\sqrt{x^2y}+15\sqrt{x^2y}[/tex]
- As you can see, the index of the radical in the first term is 2 and the index of the radical in the second term is also 2.
- You can observe that the radicands are the same.
- Therefoe, as the index of the first radical is equal to the index of the second radical and the radicands are the same, you can make the addition. Then, you obtain:
[tex]35\sqrt{x^2y}[/tex]
THE ANSWER IS C all the other people are wrong because they don’t even got the answer choices. 7(5sqrt ^2y)
Which answer describes the function f(x)=2x^3-x^2
Odd
Even
Neither
it is neither, it cannot be raised to a power unless it is 0 or 1 :)
The function is neither an odd function nor an even function
The function is given as:
f(x)=2x^3-x^2
Calculate f(-x)
f(-x)= 2(-x)^3 - (-x)^2
f(-x)= -2x^3 - x^2
Calculate -f(x)
-f(x)= -2x^3 + x^2
From the above computation,
f(x), f(-x) and -f(x) are not equal
Hence, the function is neither an odd function nor an even function
Read more about functions at:
https://brainly.com/question/17059941
the functuion of g(x)= 2x-3 is compressed horizontally to form f(x). which of the following could be the equation of f(x) ?
A.f(x)=1/3x-1
B.f(x)=x+12
C.f(x)=-2x+1
D.f(x)=4x-3
Answer:
Please see attached graph for further understading of the solution.
To easily solve this problem, we can graph each of the functions and compare them to the original one.
g(x)= 2x-3
An horizontal compression implies a greater slope for the line. This means that the graph of f(x), must reach higher values faster.
Based on the image, the answer is option D
A bookstore is selling books for $10 each which graph shows the relationship between the number of books x the store sold and the total amount of money y paid from the book sales
Answer:
one or two books cost 10 dollars
Step-by-step explanation:
which graph shows the relationship between the number of books x the store sold and the total amount of money y paid from the book sales?
book store selleing books for
10 dollars
x= books 1,2
y= paid 10
so u see it says someone bought books for 10 dollars
each so two books
Answer with explanation:
Selling Price of book, which the bookstore is selling =$ 10
If, total number of books sold =x
Total Selling Price of x, books = $10 × x=$10 x
If, y is the amount of money obtained after selling x books, then
→→ y= 10 x
Drawing the graph on "Desmos graphing"
Rewrite 11/35 and 3/10 so they have common denominator
Final answer:
To rewrite 11/35 and 3/10 with a common denominator, find the LCD, which is 70 and it will result in 22/70 and 21/70.
Explanation:
To rewrite the fractions 11/35 and 3/10 so they have a common denominator, we need to find the Least Common Denominator (LCD) for both fractions. The denominators are 35 and 10. Prime factorizing 35, we get 5 and 7. Prime factorizing 10, we get 2 and 5.
The LCD for both will therefore be the product of these prime factors taken at the highest power that appears in each, which is 2 * 5 * 7 = 70. To rewrite the fractions, we multiply the numerator and the denominator of each fraction by any factors needed to reach the LCD.
For 11/35 to have 70 as its denominator, we observe that 35 already has the factor 7 and 5, hence it only needs to be multiplied by 2. So, 11/35 = 11 * 2 / 35 * 2 = 22/70.
For 3/10 to have 70 as its denominator, we see that 10 has the factor 2 and 5, so it needs to be multiplied by 7. Therefore, 3/10 = 3 * 7 / 10 * 7 = 21/70.
Now both fractions have the same denominator of 70, which is 22/70 and 21/70 respectively.
orlando has a bag of 37 apples and a bag of 29 apples he can bake 6 apples in a pan how many pans can orlando make. plzzzzzzz i need help now and thx
the answer is 11 trays
What number am I ???
Answer:
17
Step-by-step explanation:
Since the number has only 2 factors it must be a prime number
A prime number has 2 factors 1 and itself
Since the difference between the factors is 16 then
17 - 1 = 16 and so the number is 17 with factors 1 and 17
simplify 4x2 + 8x - 11x + 6 - 5x2 + 2
Answer:
[tex]\boxed{\bold{-5x+8}}[/tex]
Step-by-step explanation:
Combine Similar Elements: [tex]\bold{8x-11x=-3x}[/tex]
[tex]\bold{4\cdot \:2x-3x+6-5\cdot \:2x+2}[/tex]
Multiply: [tex]\bold{4\cdot \:2=8}[/tex]
[tex]\bold{8x-3x+6-10x+2}[/tex]
Group Like Terms
[tex]\bold{8x-3x-10x+6+2}[/tex]
Combine Similar Elements: [tex]\bold{8x-3x-10x=-5x}[/tex]
[tex]\bold{-5x+6+2}[/tex]
Add Numbers: [tex]\bold{6+2=8}[/tex]
[tex]\bold{-5x+8}[/tex]
To simplify the expression, combine like terms resulting in -1[tex]x^2[/tex] - 8x + 8.
To simplify the given algebraic expression, we need to combine like terms. The terms in the expression are: 4[tex]x^2[/tex], 8x, -11x, 6, -5[tex]x^2[/tex], and 2. Combining the [tex]x^2[/tex] terms and the x terms separately, we get:
[tex]4x^2 - 5x^2 = -1x^2[/tex]
8x - 11x - 5x = -8x
Finally, we add the constant terms: 6 + 2 = 8.
Putting it all together, the simplified version of the expression is: -1[tex]x^2[/tex] - 8x + 8.
Solve the system of equations 3x + y = 3 and 7x + 2y = 1
Answer: x=-5 and y=18
Step-by-step explanation:You multiply 3x+y=3 by 2 to get 6x+2y=6. Then, you subtract 6x+2y=6 from 7x+2y=1. The result is x=-5. You then plug -5 into an x in one of the equations and get y equal to 18.
Answer:
-5
Step-by-step explanation:
If BC=6.7, CD=6.2, and AD=13.3, find AB to the nearest tenth
Answer:
14.4
Step-by-step explanation:
AB / BC = AD/CD
AB/6.7 = 13.3 / 6.2
AB = 13.3 * 6.7 / 6.2
AB = 14.4
Answer:
The answer is 14.4 units.(option A)
Step-by-step explanation:
Angles BAC and CAD are equal.
We can solve this by using the proportional relation.
[tex]\frac{AB}{BC} =\frac{AD}{CD}[/tex]
Let AB be = x
So, [tex]\frac{x}{BC} =\frac{AD}{CD}[/tex]
[tex]\frac{x}{6.7} =\frac{13.3}{6.2}[/tex]
[tex]\frac{x}{6.7} =2.145[/tex]
[tex]x=6.7\times2.145[/tex] = 14.37 units
And rounding to nearest tenth, we get 14.4 units.
Order the ratios from least to greatest. 5:3, 6:4, 11/3, 3 to 1, 33:100
Answer:
33:100- 6:4- 5:3- 11/3
Step-by-step explanation:
First turn into even fractions
5:3= 1 2/3
6:4=1 1/2
11/3= 3 2/3
3/1= 3
33:100= 1/3
Then look at the numbers.
Can someone please answer the chart part please?? ASAP!!
When x=0, that’s the y-intercept. So in this case the y-intercept would be (0,2)